tan ydx - x ln xdy=0

Answers

Answer 1

Answer:

Uuuzu7ggijjrudidjdiwisiwiwieie


Related Questions

For the diagram below, which of the following represents the length of line MN to the nearest tenth?

Answers

Answer:

( 4 ) 28.6Step-by-step explanation:

Tan 40 = 24/MNMN = 24/tan 40

HOPE IT HELPS !!!!

Trigonometry help please? I need the three answers

Answers

Answer:

Both triangles are triangle rectangles, but the triangles are not similar.

Step-by-step explanation:

By the Pythagorean's theorem, we know that for a triangle rectangle the sum of the squares of the cathetus is equal to the square of the hypotenuse.

Where the cathetus are always the two sides of smaller length.

We also know that two figures are similar if all the correspondent sides are proportional to each other, this means that the figures have the same shape but different size.

First, for triangle A the measures of the sides are:

48, 55, 73

Here the two catheti are 48 and 55, and the hypotenuse is 73.

Then to answer the first question we need to try to apply the Pythagorean's theorem, we should have:

48^2 + 55^2 = 73^2

solving that we get:

5,329 = 5,329

This is true, thus triangle A is a triangle rectangle.

Now for triangle B the measures are: 36, 77 and 85.

So the catheti are 36 and 77, and the hypotenuse is 85

So to check if triangle B is a triangle rectangle the equation:

36^2 + 77^2 = 85^2

must be true, solving both sides we get:

7,225 = 7,225

This is true, so triangle B is a triangle rectangle.

Finally, to check if the figures are similar we need to compare the correspondent sides of both triangles, such that the quotient of correspondent sides must be always the same.

For the hypotenuses, if we compute:

(hypotenuse B)/(Hypotenuse A) we get:

85/73 = 1.16

Now if we do the same for the two smaller catheti we get:

36/48 = 0.75

The quotients are different, thus the triangles are not similar.

Which table represents a linear function?

Answers

Answer:

First table from left.

Step-by-step explanation:

Equal step of a half.

1,7

2,9

3,13

4,21

Step-by-step explanation:This is the linear function graph i plotted the points

How much is 12,856 ounces in pounds ?

Answers

Answer : 803.5 pounds

There are 16 ounces in a pound, so just divide 12,856 by 16, giving you 803.5.

G.1.- Una Recta contiene los puntos (-3,7)
y (9,-5) Calcule la ecuación de la recta en la
FORma y=mxtb. Explicar los pasos

Answers

Given:

A line passes through the points (-3,7) and (9,-5).

To find:

The equation of the line in the form of [tex]y=mx+b[/tex].

Solution:

If a line passes through two points, then the equation of the line is:

[tex]y-y_1=\dfrac{y_2-y_1}{x_2-x_1}(x-x_1)[/tex]

A line passes through the points (-3,7) and (9,-5). So, the equation of the given line is:

[tex]y-7=\dfrac{-5-7}{9-(-3)}(x-(-3))[/tex]

[tex]y-7=\dfrac{-5-7}{9+3}(x+3)[/tex]

[tex]y-7=\dfrac{-12}{12}(x+3)[/tex]

[tex]y-7=-1(x+3)[/tex]

On further simplification, we get

[tex]y-7=-x-3[/tex]

[tex]y-7+7=-x-3+7[/tex]

[tex]y=-x+4[/tex]

Therefore, the equation of the required line is [tex]y=-x+4[/tex].

Find the common ratio of the geometric sequence − 1 , − 9 , − 81 ,

Answers

Step-by-step explanation:

everything can be found in the picture

please help me please help me please help me please help me please help me please help me please​

Answers

Answer:

1. -4

2(12,35,37). hope helpful answer

Answer:

Question 1 = 256

Question 2 = ( 7, 8, 12)

A) work out the value of g.
Give your answer in standard form correct to three significant figures.
B) work out the new value of g. Give your answer in standard form correct to 3 significant figures. (M is increased by 8% and T is increased by 5%).

Answers

Answer:

4547.14

Step-by-step explanation:

m increased by %8 so it'll be

[tex]6.588 \times {10}^{ - 5} [/tex]

and t will be

[tex]1.785 \times {10}^{ - 6} [/tex]

so G =

[tex] \sqrt{ \frac{(6.588 \times {10}^{ - 5}) }{ {(1.785 \times {10}^{ - 6}) }^{2} } } [/tex]

G= 4547

What is the value of x in the equation 1/5x-2/3y = 30, when y = 15

Answers

Answer:

x=200

Step-by-step explanation:

(1/5)x-(2/3)y=30, y=15

(1/5)x-(2/3)*15=30

(1/5)x=40

x=200

Rounding in the calculation of monthly interest rates is discouraged. Such rounding can lead to answers different from those presented here. For long-term loans, the differences may be pronounced.

Assume that you take out a $2000 loan for 30 months at 8% APR. How much of the first month's payment is interest? (Round your answer to the nearest cent.)
$

Answers

Answer:

13.33

Step-by-step explanation:

Because it's only payment one we can easily figure out the interest portion

we first need the effective rate

.08/12= .006666667

We the just mulitply this by loan amount

2000*.006666667= 13.33

Step-by-step explanation:

its your answer

I hope it's help you

Help me please with this math problem

Answers

Answer:

[tex]x=14[/tex]

Step-by-step explanation:

[tex](5x-14)+(8x+12)=180[/tex]

These two angles on the line is 180°

Solve the equation:

[tex]5x-14+8x+12=180[/tex]

[tex]5x+-14+8x+12=180[/tex]

[tex](5x+8x)+(-14+12)=180[/tex] {combine the like terms}

[tex]13x-2=180[/tex]

[tex]13x=180+2[/tex]

[tex]13x=182[/tex]

[tex]x=182/13[/tex]

[tex]x=14[/tex]

PROOF:

{substitute 14 in the place of x}

[tex](5(14)-14)+(8(14)+12)=180[/tex]

[tex]56+124=180[/tex]

[tex]180=180[/tex]

hope this helps....

solve the paper with the correct answer​

Answers

Answer:

I did question no 2 , 4 , 5 , 6

from question attempt any two questions i did 4 and 5.

Step-by-step explanation:

Hope this helps u !!

What is the generalization of a point which is
reflected over the x axis?

Answers

Answer:

If a point is reflected over the x-axis, the x- coordinate of the image remains the same as the pre-image and the y- coordinate is the opposite of the pre-image. ... If a point is reflected over both axes, both coordinates of the image are opposite of the pre-image.

Anyone knows what the answer for this is I am really confused on it

Answers

By replacing x in the equation with -1 our result will be 1.5. So f(-1)=1.5


Please answer part A and B.

Answers

Answer:

Percent who walk 45%

Percent in car 22.5%

Step-by-step explanation:

The total number of students is

36+26+18 = 80

Percent who walk is 36/80 = .45 =45%

Percent in car is 18/80 =.225= 22.5%

What are the coordinates of the vertex of the parabola described by the
equation below?
y= 2x+52-3
O A (-5.3)
0
B. (-3.-5)
C. (3.5)
O D. (5-3)
ANSWER ASAP!

Answers

Step-by-step explanation:

please type the question properly

Did you mean y = (2x +5)^2 - 3 ?
The answer would then be (-2.5, -3)

Calculate the mean, the variance, and the standard deviation of the following discrete probability distribution. (Negative values should be indicated by a minus sign. Round intermediate calculations to at least 4 decimal places. Round your final answers to 2 decimal places.)
x −36 −26 −15 −4
P(X = x) 0.32 0.36 0.21 0.11
Mean
Variance
Standard deviation

Answers

Answer:

Mean = 24.47

Variance = 108.31

Standard deviation = 10.41

Step-by-step explanation:

The probability distribution table has been attached to this response.

(1) To calculate the mean (m)

(a) First multiply each of the values of x by their corresponding probability values.

This is shown in the third column of the table.

(b) The sum of the results in the third column gives the mean of the distribution. i.e

m = ∑xP(x) = 11.52 + 9.36 + 3.15 + 0.44

m = 24.47

(2) To calculate the variance (σ²).

(a) First find the square of the difference between the values of x and the mean (m) calculated in (1b) above. i.e

(x - m)²

The result is shown in the fourth column of the table.

(b) Next, multiply each of the results in the fourth column (x - m)², by their corresponding probability values P(X = x). i.e

(x - m)²(P(X = x))

The result is shown in the fifth column of the table.

(c) Now find the variance (σ²) which is the sum of the results in the fifth column. i.e

σ² = ∑(x - m)²(P(X = x)) = 42.5411 + 0.8427 + 18.8330 + 46.0923

σ² = 108.3091

σ² = 108.31      [to 2 decimal places]

(3) To calculate the standard deviation (σ)

The standard deviation is the square root of the variance of the distribution. Calculate this by finding the square root of the result in (2c) above.

σ = √σ²

σ = [tex]\sqrt{108.31}[/tex]

σ = 10.4072

σ = 10.41        [to 2 decimal places]

25^2 6^3 find prime factorization

Answers

Answer:

With Exponents: 32 × 7 × 401

Without Exponents: 3 × 3 × 7 × 401  

Step-by-step explanation:

Hello,

[tex]25^2*6^3\\\\=(5^2)^2*(2*3)^3\\\\=\boxed{2^3*3^3*5^4}\\[/tex]

Inscribe a circle in a unit square and to r= 1000 random.

a. Suppose that 760 of those darts land in the circle.
b. What would our estimate be if we let n â oo and we applied the same ratio strategy?

Answers

Step-by-step explanation:

area of square = 1

area of a circle in square = [tex]\pi r^2 = \pi *(1/2)^2 = \frac{\pi }{4}[/tex]

bu estimate  [tex]p^ = X/n = 760/1000 = 0.76

Therefore,

[tex]\frac{\pi }{4} \approx 0.76[/tex]

[tex]\pi \approx 0.76*4 =3.04[/tex]

b) if n tend to infinity

[tex]\pi will approach 3.14[/tex]

Answer:

uvfhmfa

Step-by-step explanation:

I need help with the problem

Answers

Answer:

Non-linear

Step-by-step explanation:

While the difference in x terms remains constant at 1

The difference between y terms varies.

Answer:

Non-linear.

Step-by-step explanation:

Solve for T: 10t-4x=3S Explanation plz

Answers

T=3s/10+2x/5 hope this helped <3

The mean price for a 2,000sq foot home in FL is $240,000 with a Standard Deviation of $16,000. The mean price of the same sized home in OH is $170,000 with a standard deviation of $12,000. Which state would a home priced at $200,000 be closer to the mean price, compared to the distribution of prices in the state?
Find the z score for each state.

Answers

Answer:

The z-score for a home priced at $200,000 in Florida is of -2.5.

The z-score for a home priced at $200,000 in Ohio is of 2.5.

The closeness to the mean is measured by the absolute z-score(disconsidering the signal, the lower the score, the closer to the mean). However, in this case, both z-scores have the same absolute value, so in both Florida and Ohio a home priced at $200,000 is equally close to the mean.

Step-by-step explanation:

Z-score:

In a set with mean [tex]\mu[/tex] and standard deviation [tex]\sigma[/tex], the z-score of a measure X is given by:

[tex]Z = \frac{X - \mu}{\sigma}[/tex]

The Z-score measures how many standard deviations the measure is from the mean. After finding the Z-score, we look at the z-score table and find the p-value associated with this z-score. This p-value is the probability that the value of the measure is smaller than X, that is, the percentile of X. Subtracting 1 by the p-value, we get the probability that the value of the measure is greater than X.

The mean price for a 2,000sq foot home in FL is $240,000 with a Standard Deviation of $16,000. Home of $200,000.

This means that [tex]\mu = 240, \sigma = 16, X = 200[/tex]. So

[tex]Z = \frac{X - \mu}{\sigma}[/tex]

[tex]Z = \frac{200 - 240}{16}[/tex]

[tex]Z = -2.5[/tex]

The z-score for a home priced at $200,000 in Florida is of -2.5.

The mean price of the same sized home in OH is $170,000 with a standard deviation of $12,000. Home of $200,000.

This means that [tex]\mu = 170, \sigma = 12, X = 200[/tex]. So

[tex]Z = \frac{X - \mu}{\sigma}[/tex]

[tex]Z = \frac{200 - 170}{12}[/tex]

[tex]Z = 2.5[/tex]

The z-score for a home priced at $200,000 in Ohio is of 2.5.

Which state would a home priced at $200,000 be closer to the mean price, compared to the distribution of prices in the state?

The closeness to the mean is measured by the absolute z-score(disconsidering the signal, the lower the score, the closer to the mean). However, in this case, both z-scores have the same absolute value, so in both Florida and Ohio a home priced at $200,000 is equally close to the mean.

Hall and Mindy are playing a guessing game. Hall tells Mindy: ""The difference between 17 and the square root of my mystery number is 5"". What are two possible numbers that Hall could be thinking of?

Answers

Answer:

The possible number that Hall could be thinking of is 144.

Step-by-step explanation:

Difference between 17 and the square root of my mystery number

This is represented by:

[tex]17 - \sqrt{x}[/tex]

is 5

Then:

[tex]17 - \sqrt{x} = 5[/tex]

What are two possible numbers that Hall could be thinking of?

We have to solve the equation, for x. So

[tex]17 - \sqrt{x} = 5[/tex]

[tex]\sqrt{x} = 12[/tex]

[tex](\sqrt{x})^2 = (12)^2[/tex]

[tex]x = 144[/tex]

The possible number that Hall could be thinking of is 144.

When solving inequality

If in the numerator i had a imaginary number when factoring what should i do???

Answers

Answer:

we need to factorise if imaginary number is in the denominator. no action is required if its in the numerator.

you can factorise an imaginary number by multiplying both the numerator and denominator by the conjuncate of the complex number

for instance,

we've given a complex number as follows

[tex] \frac{x}{3x + 4i} [/tex]

factorising

[tex] \frac{x}{3x + 4i} \times \frac{3x - 4i}{3x - 4i} [/tex]

as 3x -4i is the conjugate of 3x + 4i

and 4i is the imaginary number

[tex] \frac{ x(3x - 4i)}{9x {}^{2} - 4 {i}^{2} }[/tex]

and since i² = -1

[tex]\frac{ x(3x - 4i)}{9x {}^{2} + 4}[/tex]

hence factorised

Show number that 0.245 is Rational number

Answers

Answer:

A rational number is one that may be represented in the manner p/q.

245/1000 = 0.245

Step-by-step explanation:

0.245 is not an integer. It is a rational number.

Find the value of x?

Answers

Answer:

X=2.

Step-by-step explanation:

3 times 2 is equal to 6, and 4 times 2 is equal to 8, 6+8=14.

A system of equations is shown below:

y = 3x − 7
y = 2x + 1

What is the solution to the system of equations?

(8, 17)
(−8, 17)
(−8, −17)
(8, −17)

Answers

Answer:

(8, 17)

Step-by-step explanation:

y= 3x -7 -----(1)

y= 2x +1 -----(2)

Substitute (1) into (2):

3x -7= 2x +1

Being x terms to one side, constant to the other:

3x -2x= 7 +1

x= 8

Substitute x= 8 into (2):

y= 2(8) +1

y= 16 +1

y= 17

∴ The solution is (8, 17).

help me find answer i need answer please hlep

Answers

Answer:

7

Step-by-step explanation:

This is correct answer bro...

Answer:

7

Step-by-step explanation:

So first off, two numbers that are to the same root can be combined.

In this case we can combine these two values, since they are to the 5th root and are both 7:

[tex]\sqrt[5]{7*7^4}[/tex]

=

[tex]\sqrt[5]{49^4}[/tex]

This can be broken down into 7^2

This gets us [tex]\sqrt[5]{7^5}[/tex]

Remember that we add exponents, together, so in this case it adds up to 5.

You might be wondering...well if its 7^2 and 7^4, wouldnt it be 7^6?

Well, remember this. One of the 7s in 7^2 is from the 7^4, so tecnically it is 7^3 + 7^2, which is 7^5

Anyway, now we have:

[tex]\sqrt[5]{7^5}[/tex]

When you root a exponent, it works like subtraction.

In this case we have a exponent of 5, and a root of 5.

This is 5-5 = 0

So it will just be 7. Because the square and root cancel each other out, leaving us with 7.

Hope this helps!

Also, I  just watched you waste someone elses answer so you could ask your own question. Instead of wasting other peopes answers, message me on one of my questions or answers, I will help you for free. :D

Which of the following is the graph of f(x)=║x║translated 2 units right, 2 units up, and dilated by a factor of 1/3?

Answers

Answer:

c on edge2021

Step-by-step explanation:

Jesse takes two data points from the weight and feed cost data set to calculate a slope, or average rate of change. A hamster weighs half a pound and costs $2 per week to feed, while a Labrador Retriever weighs 62.5 pounds and costs $10 per week to feed. Using weight as the explanatory variable, what is the slope of a line between these two points? Answer choices are rounded to the nearest hundredth.
a. $0.13 / Ib.
b. $4.00 / Ib
c. $6.25 / Ib.
d. $7.75 / Ib.

Answers

Answer:

a. $0.13 / Ib.

Step-by-step explanation:

Slope of a line:

Suppose we have two data-points in a line. The slope is given by the change in the output divided by the change in the output.

In this question:

Input: weight(in pounds)

Output: Weekly cost to feed.

A hamster weighs half a pound and costs $2 per week to feed, while a Labrador Retriever weighs 62.5 pounds and costs $10 per week to feed.

Inputs: 0.5, 62.5

Outputs: 2, 10

Change in the outputs: 10 - 2 = 8

Change in the inputs: 62.5 - 0.5 = 62

Slope: [tex]m = \frac{8}{62} = 0.13[/tex]

So the correct answer is given by option A.

Answer:

0.13

Step-by-step explanation:

Other Questions
The following are the temperatures of 10 days in winter in C.6, 5, 3, -4, 3, -3, 2, -1, -2, 0Work out the mean temperature Determine if the graph is symmetric about the x-axis, the y-axis, or the origin.r = 4 cos 5 All mutations result in lethal diseases like Tay Sachs, sickle-cell anemia and color-blindness. Group of answer choices True or false Vivian and other members of her group are attending an event where they have to give short, uninterrupted speeches, one after the other. The moderator has a passive role during the course of the discussion, he or she has to simply introduce the presenters and manage the time frame of the event. At the end of the discussion, the moderator engages with the presenters and audience to ask questions and point to areas of agreement or disagreement.Vivian is part of a _____. Commoners elect a representative which the upper class approves of, and only the upper class can veto laws b. Upper class chooses a representative and holds more power than the commoners c. Citizens elect representatives who cast votes on their behalf, tribunes had the power to veto any laws, and all laws had to be displayed in a public place. d. A representative is chosen from both classes to represent each group equally Ryder used front end estimation to estimate the product of -24.98 - 1.29 what is the zestimate ANSWER THIS FOR 15 POINTS!!!A hot metal plate at 150C has been placed in air at room temperature. Which event would most likely take place over the next few minutes? Molecules in both the metal and the surrounding air will start moving at lower speeds. Molecules in both the metal and the surrounding air will start moving at higher speeds. The air molecules that are surrounding the metal will slow down, and the molecules in the metal will speed up. The air molecules that are surrounding the metal will speed up, and the molecules in the metal will slow down. what was Athena's physical appearance Help me solve these 4 plssss ASAP What is represented by the chemical formulaPbCl2(s)?A) a substanceB) a solutionC) a homogeneous mixtureD) a heterogeneous mixtureA hemil reation Identify whether the following transactions are primary market or secondary market transactions. a. You buy 855 shares of ABC Co. through your brokerage account. b. You buy $5,801 of XYZ Co. bonds from another investor. c. M Helppp!Is astatine (At), atomic number 85, a metal or nonmetal? Why? Determine whether the given ordered pair is a solution to the given equation. (0,-5) x+4y=-20 someone plz help me porfavor!!!!! how much greater is the volume of the refrigerator than the volume of the top part pls helppppppppppp its timedddd Suppose a nation is ruled by a single person who holds absolute power over the national government and the lives of citizens. What type of government does this nation have? are you against President Trump's insistence on the importance of building this wall. Are you for it or against it? Why or why not? Lieutenant Commander Data is planning to make his monthly (every 30 days) trek to Gamma Hydra City to pick up a supply of isolinear chips. The trip will take Data about three days. Before he leaves, he calls in the order to the GHC Supply Store. He uses chips at an average rate of seven per day (seven days per week) with a standard deviation of demand of two per day. He needs a 98 percent service probability.Required:If he currently has 35 chips in inventory, how many should he order? What is the most he will ever have to order? Frank Corporation manufactures a single product that has a selling price of $25.00 per unit. Fixed expenses total $64,000 per year, and the company must sell 8,000 units to break even. If the company has a target profit of $19,000, sales in units must be:________.a. 9,648b. 8,760c. 10,375d. 10,560